LSAT and Law School Admissions Forum

Get expert LSAT preparation and law school admissions advice from PowerScore Test Preparation.

 Administrator
PowerScore Staff
  • PowerScore Staff
  • Posts: 8916
  • Joined: Feb 02, 2011
|
#33164
Complete Question Explanation
(See the complete passage discussion here: lsat/viewtopic.php?t=13636)

The correct answer choice is (E)

This question asks for the choice that would weaken the claim that CJD is caused by prions.

Answer choice (A): The fact that viral infections may in some cases bring about the same symptoms would not undermine the claim that prions cause CJD, so this cannot be the right answer to this Weaken question.

Answer choice (B): This choice is consistent with the claim that prions cause CJD, and that the disease is inevitably fatal, so this cannot be the right answer to this Weaken question.

Answer choice (C): Even if, as this choice provides, prions can cause other types of conditions, that would not weaken the claim that they cause CJD.

Answer choice (D): The presence or absence of heredity factors would have no effect on the claim that CJD is caused by prions, so this cannot be the right answer to this Weaken question.

Answer choice (E): This is the correct answer choice. If an antibacterial drug were effective in reversing CJD, that would suggest that CJD is caused by bacteria, one of the four previously known pathogens, rather than caused by prions.
 reop6780
  • Posts: 265
  • Joined: Jul 27, 2013
|
#15575
The correct answer is E while I chose answer C.

Is answer E trying to undermine the causation between prions and CJD by showing that there might be another cause other than prions?

I do not know how exactly answer E does that. If answer E states that prions exist even though CJD disappeared, I can infer that prions are not the cause or at least not the only cause.

How exactly does E try to undermine the causation?

Also, what do I look for in science passage in general?
- STAMP VIEW? still, science passage is very different from other passages. It somehow requires more understanding of explanation.
User avatar
 KelseyWoods
PowerScore Staff
  • PowerScore Staff
  • Posts: 1079
  • Joined: Jun 26, 2013
|
#15586
Hi reop6780!

According to the passage, prions are proteins that occur naturally in the body and they are different than the pathogens that were previously thought to be the only agents responsible for infections: viruses, bacteria, fungi, and parasites.

Answer choice (E) states that a new antibacterial drug is effective at reversing CJD. If the disease responds to an antibacterial drug, this would undermine the claim that prions cause CJD by suggesting that actually there is some sort of bacteria causing CJD. Since prions are not bacteria, they would not respond to an antibacterial drug.

For science passages in general, you are looking for the same VIEWSTAMP elements that you are looking for in every other passage. They can be tricky because there is often a lot of explanation to keep track of, but this can be true of other passages as well, no matter the topic. Remember that you don't need to fully understand the "science" of the passage topic to be able to understand the passage.

Hope this helps!

Best,
Kelsey
 reop6780
  • Posts: 265
  • Joined: Jul 27, 2013
|
#15596
ah ha! "antibacterial" !

Thank you!
 Brandonhsi
  • Posts: 16
  • Joined: Jul 12, 2014
|
#18862
(C): I believe (C) is saying that later research links prions to degenerative conditions NOT affecting the brain or other central nervous system.

If it shows prions not affecting other systems, it can weaken the claim prions cause CJD?

Thanks,
Brandon
 Lucas Moreau
PowerScore Staff
  • PowerScore Staff
  • Posts: 216
  • Joined: Dec 13, 2012
|
#18863
Hello, Brandon,

Answer choice C is saying exactly that, yes. And that is why it's not the correct answer choice. ;)

If C was true, that wouldn't affect the claim that prions cause CJD, neither weakening it nor strengthening it. At the very least, it would weak the claim that prions only cause CJD, but, that's not what was asked.

Answer choice E is correct, because an antibacterial drug wouldn't affect prions - prions are not bacteria! If an antibacterial drug reverses the effect of CJD, that indicates strongly that something other than prions cause CJD, and as such, it would weaken the claim that prions cause CJD.

Hope that helps,
Lucas Moreau
 harvoolio
  • Posts: 63
  • Joined: Apr 25, 2018
|
#45195
Is another reason why (E) is correct because of the fatal aspect?

So, assume answer choice (E) did not have "antibacterial" would it still be correct because paragraph (3) established that prions are presently unstoppable or is this going to far? In other words, just because presently we cannot stop prions does not mean we will always be unable to stop them?

Thanks.
 Malila Robinson
PowerScore Staff
  • PowerScore Staff
  • Posts: 296
  • Joined: Feb 01, 2018
|
#45234
Hi Harvoolio,
In the absence of a better answer choice E might work without the antibacterial part, but it would be an extremely weak answer. The passage is saying that prions are unstoppable because science doesn't currently know how to 'treat' a normally occurring protein that the body doesn't recognize as being dangerous. That doesn't mean that there will never be a cure. It is better to attack the fact that it is prions (protein pathogens) that are causing CJD with an alternate cause. In E, it makes it likely that it is a bacterial infection that causes CJD since the antibacterial treatment is working to reverse the onset of the disease.
Hope that helps,
Malila
 harvoolio
  • Posts: 63
  • Joined: Apr 25, 2018
|
#45307
Thanks Malila.
 lsatprep1215
  • Posts: 33
  • Joined: Dec 16, 2019
|
#74621
Hi,

After reading the post I still not understand why ans C would not undermine the claim. I understand that even if it shows that prions not affecting the brain or central nervous system it could still affect other areas and thus cause CJD. But I thought base on the passage CJD only involves problems within the brain but no other area (hearts, lungs.. etc) involved. I didn't see any other area beside the brain and CNS so if it shows prions not affecting brain or CNS doesn't it weaken the claim? Can someone explain?

Get the most out of your LSAT Prep Plus subscription.

Analyze and track your performance with our Testing and Analytics Package.